Đến nội dung

Mrnhan nội dung

Có 741 mục bởi Mrnhan (Tìm giới hạn từ 27-04-2020)



Sắp theo                Sắp xếp  

#469121 Ảnh thành viên

Đã gửi bởi Mrnhan on 05-12-2013 - 21:17 trong Góc giao lưu



Nhìn mặt du côn + ngáo thế mà anh bảo handsome , cứ troll em

 

Đây là yêu cầu của em:

 

Mình đi làm tình...làm tình...tình....nguyện!

 

mrnhan.jpg

 

Chính chủ ở ngay sau Hòm Tiền ủng hộ!




#468850 Ảnh thành viên

Đã gửi bởi Mrnhan on 04-12-2013 - 21:10 trong Góc giao lưu

:luoi:  chụp mà k biết đăng ntn , ai giúp vs 

Nhìn khung soạn, nhấn vào "Sử dụng bộ soạn đầy đủ" $\to$ file đính kèm $\to$ chọn ảnh trong máy $\to$ tải lên $\to$ cho vào văn bản!!




#469128 Ảnh thành viên

Đã gửi bởi Mrnhan on 05-12-2013 - 21:23 trong Góc giao lưu

Hé hé, nhìn đầu gấu nhỉ . ĐƯợc từng ấy tiền mua kẹo cho em ăn đi :D

Tiền đó là tiền ủng hộ của các SV để hỗ trợ các SV nghèo khác!

Không có phần cho anh




#469139 Ảnh thành viên

Đã gửi bởi Mrnhan on 05-12-2013 - 21:37 trong Góc giao lưu

Hay nhỉ , hi vọng em sẽ có lần được đi làm tình nguyên, xem thử nó thế nào 

Đi làm tình...thấy vui vui! Nhưng khổ nỗi hay bị phạt..vì hay nói chuyện trong giờ họp(5k/1lan)




#469143 Ảnh thành viên

Đã gửi bởi Mrnhan on 05-12-2013 - 21:44 trong Góc giao lưu

@@~ . 5k/1 lần nữa mới chết chứ nhỉ ?? hay quá . Làm phát 500k/1 lân thì im re luôn :D

Đó là bị phạt lần đầu tiên, lần sau hình như tăng lên mỗi đợt phạt là 20k/1 lần!! nên Bị phạt rồi là im re---ko hé mồn nữa!!




#468223 Ảnh thành viên

Đã gửi bởi Mrnhan on 01-12-2013 - 21:25 trong Góc giao lưu

Post cái ảnh phát, ở giữa nha, nhìn mặt ngáo quá . !!! 

312610_186098251534084_508114572_n.jpg

 

Được đấy nhỉ, handsome đấy chứ!

Chà, khi nào anh tự sướng a sẽ up lên...




#469132 Ảnh thành viên

Đã gửi bởi Mrnhan on 05-12-2013 - 21:30 trong Góc giao lưu

 

Mà thấy ít anh nhỉ ?? Đi tình nguyện ở đâu mà được ngần đó thôi z ?? 

Đông, đây chưa đầy 1/7-1/8 đâu? mà còn phía sau nữa, bị che khuất, mấy anh đăng sau đánh đàn guitar thu hút chú ý!!




#650756 Topic về Bất đẳng thức trong Tích phân

Đã gửi bởi Mrnhan on 22-08-2016 - 10:20 trong Giải tích

Vế phải trong bất đẳng thức của bạn không biết có nhầm lẫn gì không ?

 

Hình như bạn ấy sai điều kiện thôi, điều kiện phải là $f(0)=0$

 

Nếu thế thì lời giả sẽ là:

 

$$f(x)=\int_{0}^{x}f'(t)dt\leq \left ( \int_{0}^{x}1^2 \right )^{1/2} \left( \int_{0}^{x}\left ( f'(t) \right )^2 \right )^{1/2}$$

 

$$\Rightarrow f^2(x) \leq x \int_{0}^{x}\left ( f'(t) \right )^2dt\leq x \int_{0}^{1}\left ( f'(t) \right )^2dt$$

 

$$\Rightarrow \int_{0}^{1} f^2(x)dx\leq \int_{0}^{1}xdx\int_{0}^{1}\left ( f'(t) \right )^2dt=\frac{1}{2}\int_{0}^{1}\left ( f'(x) \right )^2dx$$




#458407 Ôn thi Olympic Toán học sinh viên 2015 [Giải tích]

Đã gửi bởi Mrnhan on 18-10-2013 - 18:46 trong Thảo luận về các kì thi, các kì kiểm tra Toán sinh viên

 

 

Cùng bắt đầu với đề VMC 2013

Câu 1. Cho $x_1 = a \in \mathbb{R}$ và dãy $(x_n)$ được xác định bởi $(n+1)^2 x_{n+1} = n^2 x_n + 2n+1$. Tìm $\lim\limits_{x \to \infty} x_n$.

 

 

 

 

 

Câu 1 có nhầm không ta, $n\to +\infty $ chứ?

 

$(n+1)^2 x_{n+1} = n^2 x_n + 2n+1\Leftrightarrow \left ( n+1 \right )^2\left ( x_{n+1}-1 \right )=n^2\left ( x_{n}-1 \right )$

 

Đặt $u_{n}=n^2\:x_{n}-n^2\to u_{n+1}=u_{n}=...=u_{1}=a-1$

 

$\to x_{n}=1+\frac{a-1}{n^2}$

 

Nên $\lim_{n\to +\infty} x_{n}=1$

 

P/s: Đúng không ta?




#470454 Tổng hợp các bài toán Tích phân

Đã gửi bởi Mrnhan on 12-12-2013 - 07:41 trong Giải tích

Bài 10: ${I_{10}} = \int\limits_0^1 {{{\left( {\frac{{\ln x}}{{x + 1}}} \right)}^2}dx} $
 

 

Hình như bài 10 còn tồn đọng:

 

Giải:

 

 

 

 

$$I_{10}=\int_{0}^{1}\left ( \frac{\ln x}{1+x} \right )^2dx=\lim_{a\to 0^+}\int_{a}^{1}\ln^2xd\left (- \frac{1}{1+x} \right )dx$$

 

$$=\lim_{a\to 0^+}\left [ -\frac{\ln ^2a}{1+a}+2\int_a^1\frac{\ln x}{x(1+x)}dx \right ]$$

 

$$=\lim_{a\to 0^+}\left [ -\frac{\ln ^2a}{1+a}+2\int_a^1\frac{\ln x}{x}dx-2\int_a^1 \frac{\ln x}{1+x} dx \right ]$$

 

$$=\underbrace{\lim_{a\to 0^+}\frac{a\ln^2a}{1+a}}_{0}-2\lim_{a\to 0^+}\int_{a}^{1}\frac{\ln x}{1+x}dx$$

 

$$=-2\underbrace{\lim_{a\to 0^+}\ln a\ln(1+a)}_{0}+2\lim_{a\to 0^+}\underbrace{\int_{a}^{1}\frac{\ln(1+x)}{x}dx}_{I}$$

 

 

Đến đây thì vô hướng, chả biết làm nữa...làm liều-nghĩ theo hướng này: (!)

 

 

Ta dùng khai triên $Maclaurin:$

 

$\ln(1+x)=\sum_{i=1}^{\infty}\frac{\left ( -1 \right )^{i+1}x^i}{i}$

 

$\Rightarrow I=\int_{0}^{1}\frac{\ln(1+x)}{x}dx=\int_{0}^{1}\sum_{i=1}^{\infty} \frac{(-1)^{i+1}x^{i-1}}{i} \: dx=\sum_{i=1}^{\infty}\frac{(-1)^{i+1}}{i^2}$

 

Đang nghĩ! ~O)




#470453 Tổng hợp các bài toán Tích phân

Đã gửi bởi Mrnhan on 12-12-2013 - 06:52 trong Giải tích

Bài toán mở đầu (dễ):

Bài 1: Tính tích phân bất định sau: $$I = \int {\dfrac{1}{{{x^2}}}\sqrt {\dfrac{{x - 1}}{{x + 1}}} } dx$$

 

P.s: Giải quyết bài 1 tồn đọng:

 

Đặt $t=\sqrt{\frac{x-1}{x+1}}\Rightarrow x=\frac{1+t^2}{1-t^2}\Rightarrow dx=\frac{4t}{\left ( 1-t^2 \right )^2}\: dt$

 

 

$\Rightarrow I_1=\int \left [ \frac{\left ( 1-t^2 \right )^2}{\left ( 1+t^2 \right )^2}\: t\: \frac{4t}{\left ( 1-t^2 \right )^2} \right ]dt=\int \frac{4t^2}{\left ( 1+t^2 \right )^2}dt$

 

$=-\int 2td\left ( \frac{1}{1+t^2} \right )=-\frac{2t}{1+t^2}+\int \frac{2}{1+t^2}dt=\frac{-2t}{1+t^2}+2\arctan t+C$




#487310 Tổng hợp các bài toán Tích phân

Đã gửi bởi Mrnhan on 17-03-2014 - 00:00 trong Giải tích

Hình như bài 10 còn tồn đọng:

 

Giải:

 

 

 

 

$$I_{10}=\int_{0}^{1}\left ( \frac{\ln x}{1+x} \right )^2dx=\lim_{a\to 0^+}\int_{a}^{1}\ln^2xd\left (- \frac{1}{1+x} \right )dx$$

 

$$=\lim_{a\to 0^+}\left [ -\frac{\ln ^2a}{1+a}+2\int_a^1\frac{\ln x}{x(1+x)}dx \right ]$$

 

$$=\lim_{a\to 0^+}\left [ -\frac{\ln ^2a}{1+a}+2\int_a^1\frac{\ln x}{x}dx-2\int_a^1 \frac{\ln x}{1+x} dx \right ]$$

 

$$=\underbrace{\lim_{a\to 0^+}\frac{a\ln^2a}{1+a}}_{0}-2\lim_{a\to 0^+}\int_{a}^{1}\frac{\ln x}{1+x}dx$$

 

$$=-2\underbrace{\lim_{a\to 0^+}\ln a\ln(1+a)}_{0}+2\lim_{a\to 0^+}\underbrace{\int_{a}^{1}\frac{\ln(1+x)}{x}dx}_{I}$$

 

 

Đến đây thì vô hướng, chả biết làm nữa...làm liều-nghĩ theo hướng này: (!)

 

 

Ta dùng khai triên $Maclaurin:$

 

$\ln(1+x)=\sum_{i=1}^{\infty}\frac{\left ( -1 \right )^{i+1}x^i}{i}$

 

$\Rightarrow I=\int_{0}^{1}\frac{\ln(1+x)}{x}dx=\int_{0}^{1}\sum_{i=1}^{\infty} \frac{(-1)^{i+1}x^{i-1}}{i} \: dx=\sum_{i=1}^{\infty}\frac{(-1)^{i+1}}{i^2}$

 

Đang nghĩ! ~O)

 

Giờ mới được học làm Gamma và khai triên chuỗi Fourier:

Cách 1: Áp dụng hàm Gamma

 

Cách 2: Áp dụng khai triển Fourier

Xét hàm $f(x)=x^2, \: x\in (-\pi, \pi)$ với chu kì $T=2\pi$, ta có khai triển Fourier cho hàm tuần hoàn là

 

Đây là hàm chẵn nên $b_n=0$

$a_0=\frac{2}{\pi}\int_{0}^{\pi} f(x)dx=\frac{2\pi^2}{3}$

 

$a_n=\frac{2}{\pi}\int_{0}^{\pi}f(x)\cos nxdx=\frac{4(-1)^n}{n^2}$

 

Nên $f(x)=\frac{a_0}{2}+\sum_{n=1}^{+\infty}\left ( a_n\cos nx+b_n\sin nx \right )=\frac{\pi^2}{3}+4\sum_{n=1}^{+\infty} \frac{(-1)^n\cos nx}{n^2}=x$

 

Cho $x=0$ thì $\frac{\pi^2}{3}+4\sum_{n=1}^{+\infty} \frac{(-1)^n}{n^2}=0\Rightarrow \sum_{n=1}^{+\infty}\frac{(-1)^n}{n^2} =-\frac{\pi^2}{12}$




#470358 Tổng hợp các bài toán Tích phân

Đã gửi bởi Mrnhan on 11-12-2013 - 20:00 trong Giải tích

có 1 bài toán cũng khá được, mình là dân mới, mong các bạn và anh chị chỉ giáo. xin cám ơn
$I= \int_{0}^{3}arcsin\sqrt{\frac{x}{x+1}}dx$
$I'=\int_{1/2}^{2}(1+x-\frac{1}{x})e^{x+\frac{1}{x}}dx$

 

Cách khác:

 

Bài 2 thì thấy giải chuẩn rồi, còn bài 1 thì tôi có cách khác như sau:

 

Ta có nhận xét sau:

 

$$arc\sin\sqrt{\frac{x}{1+x}}=arc\tan\sqrt{x}$$

 

Nên:

$$I= \int arc\sin\sqrt{\frac{x}{x+1}}dx=\int arc\tan\sqrt{x}\: dx=\int arc\tan t\: d\left ( t^2 \right )$$

$$=t^2arc\tan t-\int \frac{t^2}{1+t^2}dt=\left ( t^2+1 \right )arc\tan t-t+C=\left ( x+1 \right )arc\tan\sqrt{x}-\sqrt{x}+C$$




#475802 Xét sự hội tụ của tích phân suy rộng

Đã gửi bởi Mrnhan on 06-01-2014 - 20:56 trong Giải tích

Cho thêm vài câu này, mong thread cho:

 

$5.\: \int_0^\infty\frac{\sin^2x}{x}dx$

 

$6.\: \int_1^\infty \ln^\alpha x\: \frac{\sin x}{x}dx,\: \alpha>0$

 

$7. \: \int_0^\infty \frac{\cos\alpha x}{1+x^n}dx$

 

Giải:

 

5. $I_5=\int_0^\infty \frac{\sin^2x}{x}dx=\int_0^\infty \frac{1}{2x}dx-\int_0^\infty \frac{\cos2x}{2x}dx\to \text{pk}$

 

6. Lấy $f(x)=\sin x,\: g(x)=\frac{\ln^\alpha x}{x}$

 

Vì $\int_0^\infty f(x) \: \to \text{bị chặn},\: g(x)\to \text{liên tục và tiến về 0}\to tpht$

 

7. $+\: \alpha=0\to \int_0^\infty\frac{1}{1+x^n}dx,\: \text{ht}\Rightarrow n>1$

 

$+\alpha\neq 0,\: n>0\to \int_0^{\infty}\frac{\cos\alpha x}{1+x^n} dx\:\to \text{ht, vì theo tiêu chuẩn Dirichlet}$




#475785 Xét sự hội tụ của tích phân suy rộng

Đã gửi bởi Mrnhan on 06-01-2014 - 20:26 trong Giải tích

$1.\: \int_{0}^{+\infty }\frac{ln^2xdx}{x+x^2}$

 

$2.\: \int_{0}^{+\infty }\frac{1+sinx}{lnx-3x^2}dx$

 

$3.\:\int_{0}^{+\infty }\frac{sinx}{x\sqrt{x-1}}dx$

 

$4.\:\int_{0}^{+\infty }\frac{lnx-ln(x+1))}{\sqrt{x}}dx$

 

 

Giải:

 

2. Tích phân đã cho có 2 điểm bất thường là $x=0,\: \infty$

 

$+\: 0<\alpha<1,\: x\to 0:\: \lim_{x\to 0} x^\alpha\: \frac{1+\sin x}{\ln x-3x^2}=\lim_{x\to 0}\frac{x^\alpha}{\ln x-3x^2}=0$

 

$+\: 1<\beta<2,\: x\to \infty:\: \lim_{x\to \infty} x^\beta\frac{1+\sin x}{\ln x-3x^2}=0$

 

$\to$ tích phân HT

 

3. 

 

4. Tích phân có 2 điểm bất thường là $x=0,\: \infty$

 

$+\: \frac{1}{2}<\alpha<1:\: \lim_{x\to 0} x^\alpha \frac{\ln x-\ln (1+x)}{\sqrt{x}}=0$

 

$+\: 1<\beta<\frac{3}{2}:\: \lim_{x\to \infty}x^\beta\frac{\ln x-\ln(1+x)}{\sqrt{x}}=\lim_{x\to \infty} x^{\beta-\frac{1}{2}}\ln\left ( \frac{x}{1+x} \right )=0$

 

$\to$ tích phân HT




#475622 Xét sự hội tụ của tích phân suy rộng

Đã gửi bởi Mrnhan on 05-01-2014 - 21:59 trong Giải tích

$1.\: \int_{0}^{+\infty }\frac{ln^2xdx}{x+x^2}$
$2.\: \int_{0}^{+\infty }\frac{1+sinx}{lnx-3x^2}dx$

$3.\:\int_{0}^{+\infty }\frac{sinx}{x\sqrt{x-1}}dx$

$4.\:\int_{0}^{+\infty }\frac{lnx-ln(x+1))}{\sqrt{x}}dx$

 

Giải:

 

$1.\: \int_{0}^{+\infty }\frac{\ln^2x}{x+x^2}dx$

 

Tích phân có 2 điểm bất thường $x=0, \: \infty$

 

$+\: \text{khi}\: x\to 0:\: \frac{\ln^2x}{x+x^2}\sim\frac{\ln^2x}{x},\: \to \text{pk}$

 

Xét 1 điểm mà thấy phân kỳ rồi là cả tích phân nó phân kỳ luôn rôi!! ~O)

 

Câu 3 có vấn đề không??




#569625 THƯ MỜI HỌP MẶT 5/7/2015 tại Tp. Hồ Chí Minh

Đã gửi bởi Mrnhan on 03-07-2015 - 11:10 trong Góc giao lưu

Khi nào mới tổ chức ở Hà Nội thế BQT :)




#478146 giúp em bài tập ma trận chuyển cơ sở

Đã gửi bởi Mrnhan on 19-01-2014 - 22:36 trong Đại số tuyến tính, Hình học giải tích

Trên $\mathbb{R}^3$ cho $B$ là cơ sở chính tắc, và

$S = \{a_1 = (1,-2,3), a_2 = (4,1,-2), a_3 = (3,0,1)\}$
$T = \{b_1 = (-2,3,5), b_2 = (6,-1,4), b_3 = (7,3,-2)\}$
Viết các ma trận chuyển cơ sở P$(B \to S)$ và P$(B \to T)$ để suy ra P$(S \to T)$.
 

 

Giải:

 

Cơ sở chính tắc $\mathbb{R^3}$ là:

 

$B=\left \{ e_1=(1,0,0),e_2=(0,1,0),e_3=(0,0,1) \right \}$

 

Ma trận chuyển cơ sở từ $B\to S$

 

$$\left\{\begin{matrix}a_1=e_1-2e_2+3e_3\\a_2=4e_1+e_2-2e_3\\a_3=3e_1+0e_2+e_3 \end{matrix} \right.\Rightarrow \left [ u \right ]_B^S=\begin{pmatrix} 1&4&3\\-2&1&0\\3&-2&1\end{pmatrix}$$

 

Tương tự ma trận chuyển cở sở từ $B\to T$

 

$$\Rightarrow [u]_B^T=\begin{pmatrix} -2&6&7\\3&-1&3\\5&4&-2\end{pmatrix}$$

 

Ma trận chuyển cơ sở từ $S\to T$

 

$$\Rightarrow [u]_S^T=\left ( [u]_B^S \right )^{-1}[u]_B^T=\begin{pmatrix} 1&4&3\\-2&1&0\\3&-2&1\end{pmatrix}^{-1}\begin{pmatrix} -2&6&7\\3&-1&3\\5&4&-2\end{pmatrix}=...$$

 

Kết quả nhân 2 ma trận với nhau(ngại tính:))): Ma trận




#447643 ĐẠO HÀM- VI PHÂN- TÍCH PHÂN- ỨNG DỤNG

Đã gửi bởi Mrnhan on 03-09-2013 - 22:37 trong Giải tích Toán học

                                                       logo8namvmf.jpg

 

 

                                                   CHƯƠNG I: ĐẠO HÀM- VI PHÂN- ỨNG DỤNG

   

 

   1. Định nghĩa- Tính chất

   

   1.1. Đạo hàm

Cho hàm số $y=f(x)$ xá định trong miền $X$, đạo hàm của hàm $f(x):$

$y'=f'(x)=\lim_{\Delta x\to 0}\frac{\Delta y}{\Delta x}=\lim_{\Delta x\to 0}\frac{f(x+\Delta x)-f(x)}{\Delta x},$ $x\, \epsilon \, X$

 

Đạo hàm bên phải: $f'(x^+)=\lim_{\Delta x\to 0^+}\frac{\Delta y}{\Delta x}$

Đạo hàm bên trái: $f'(x^-)=\lim_{\Delta x\to 0^-}\frac{\Delta y}{\Delta x}$

Về hình học: $f'(x_0)$ là hệ số góc của tiếp tuyến với đồ thị của hàm số tại $(x_0, f(x_0))$.

   

   1.2. Vi phân

$y=f(x)$ gọi là khả vi tại $x\, \epsilon \, X$ $\Leftrightarrow dy=f'(x)dx\Leftrightarrow \frac{dy}{dx}=f'(x)$

   

   1.3. Tính chất

1.3.1 $f(x)$ khả vi tại $x\Leftrightarrow \exists\: f'(x)$

1.3.2 $f(x)$ khả vi tại $x\Rightarrow f(x)$ liên tục tại $x$

1.3.3 $f(x)$ khả vi tại $x$ và đạt cực trị tại $x\Rightarrow f'(x)=0$

   

   1.4 Đạo hàm và vi phân cao cấp

Định nghĩa: $y=f(x)$, $y^{(n)}=(y^{n-1})', \: \frac{d^ny}{dx^n}=\frac{d}{dx}(\frac{d^{n-1}y}{dx^{n-1}})$, $n\, \epsilon \, \mathbb{N}^*$

Quy tắc tính: 

$$(u+v)^{(n)}=u^{(n)}+v^{(n)}$$

$$(uv)^{(n)}=\sum_{k=0}^{n}C_{n}^{k}\, u^{(n-k)}\, v^{(k)}$$

   

   1.5. Công thức thông dụng

$+\: (x^\alpha )^{(n)}=\alpha (\alpha -1)(\alpha -2)...(\alpha -n+1)x^{\alpha -n}$

 

$+\: (sinx)^{(n)}=sin(x+\frac{n\pi}{2})$

 

$+\: (cosx)^{(n)}=cos(x+\frac{n\pi}{2})$

 

$+\: (a^x)^{(n)}=a^x(lna)^n$

 

$+\: (log_ax)^{(n)}=\frac{(-1)^{n-1}(n-1)!}{lna.x^n}$

 

$+\: (lnx)^{(n)}=\frac{(-1)^{n-1}(n-1)!}{x^n}$

 

   2. CÁC ĐỊNH LÝ VỀ HÀM KHẢ VI

   

   2.1 Các định lý trung bình

Định lý Rolle 

Nếu $f(x)$ liên tục trên đoạn $[a;b]$, khả vi trong khoảng $(a;b)$ và $f(a)=f(b)$ thì $\exists c\: \epsilon \:(a;b):\: f'(c\: )=0$

Bài tập vận dụng:

$*)\: \left\{\begin{matrix}a_0 \neq 0\\a_0+\frac{a_1}{2}+....+\frac{a_{n}}{n+1}=0 \end{matrix}\right.$

CMR: PT $f(x)=a_0+ a_1x+...+a_nx^n=0$ có nghiệm trong khoảng $(0;1)$

Giải:

Nguyên hàm của $f(x): F(x)=a_0x+\frac{a_1x^2}{2}+....+\frac{a_{n}x^{n+1}}{n+1}$  liên tục trên đoạn $[0;1]$, khả vi trong khoảng $(0;1)$

Dễ thấy $F(0)=F(1)\to \: f(x)=0$ có nghiệm thuộc $(0;1)$

 

Định lý Lagrange

Nếu $f(x)$ liên tục trên đoạn $[a;b]$, khả vi trong khoảng $(a;b)$ thì $\exists c\: \varepsilon \: (a;b): \: f(b)-f(a)=(b-a)f'(c\: )$

Bài tập Vận dụng:

a) Cho $m>0$ và 3 số thực $a,\:b, \: c$ tùy ý thỏa mãn: $$\frac{a}{m+2}+\frac{b}{m+1}+\frac{c}{m}=0$$

CRM: PT $ax^2+bx+c=0$ có ít nhất 1 nghiệm thuộc khoảng $(0;1)$

Giải:

Xét hàm số: $f(x)=\frac{ax^{m+2}}{m+2}+\frac{bx^{m+1}}{m+1}+\frac{cx^m}{m}, \:\:m>0$ liên tục trên đoạn $[0;1]$, khả vi trong khoảng $(0;1)$

Ta có: $f'(x)=ax^{m+1}+bx^{m}+cx^{m-1}$

Theo định lý Lagrange, tồn tại $x_0\:\epsilon \:(0;1)$ sao cho:

$\frac{f(1)-f(0)}{1-0}=f'(x_0)=0\Leftrightarrow x_0^{m-1}(ax^2+bx+c)=0$

Mà $x_0\:\epsilon \:(0;1)$ nên $ax_0^2+bx_0+c=0$

Vậy phương trình $ax^2+bx+c=0$ có ít nhất 1 nghiệm thuộc khoảng $(0;1)$

b) Chứng minh bất đẳng thức sau:

$|sina-sinb|\leq |a-b|$

Giải:

Xét hàm số $f(x)=sinx$ thì $f(t)$  thỏa mãn mọi điều kiện của định lý Lagrange trong đoạn $[a;b]$

Vậy $f(a)-f(b)=f'(c\: )(a-b)$$\Leftrightarrow sina-sinb=(a-b)cosc\Leftrightarrow |sina-sinb|=|(a-b)cosc|\leq |a-b|$ vì |cosc\leq 1|

 

Định lý Cauchy

Nếu $f(x), \: g(x)$ liên tục trên đoạn $[a;b]$, khả vi trong khoảng $(a;b)$ và $g'(x)\neq 0$ trong $(a;b)$ thì $\exists c\: \epsilon \: (a;b):\: \frac{f(b)-f(a)}{g(b)-g(a)}=\frac{f'(c\: )}{g'(c\: )}$

Bài tập Vận dụng:

Cho hàm $f(x)=x^2,\: g(x)=x$ trong đoạn $[-1;1]$

Giải:

Hàm $f(x),\: g(x)$ liên tục trong $[-1;1]$ và khả vi trong khoản $(-1;1)$ và $g'(x)\neq 0$ trong $(-1;1)$

Áp dụng định lý Cauchy

$\frac{f(1)-f(-1)}{g(1)-g(-1)}=\frac{f'(c\: )}{g(c\: )}\Leftrightarrow \frac{0}{2}=\frac{2c}{1}\Leftrightarrow c=0$

 

Định lý L'Hôpital ( khử dạng vô định $\frac{0}{0}, \: \frac{\infty }{\infty }$)

Nếu $f(x), \: g(x)$ thỏa mãn các điều kiện của định lý Cauchy trong lân cận của $x_0\:\epsilon \:\mathbb{R}^{~}$, trừ tại $x_0$ $\lim_{x\to x_0 }f(x)=\lim_{x\to x_0 }g(x)=0 \:  (\infty )$ và $\lim_{x\to x_0 }\frac{f'(x)}{g'(x)}=a\: (a\: \epsilon \: \mathbb{R}^{~})$ thì $\lim_{x\to x_0 }\frac{f(x)}{g(x)}=a$

 Bài tập Vận dụng:

Tìm giới hạn:

$$\lim_{x\to 0}\frac{tgx-x}{x-sinx}$$

Giải:

$\lim_{x\to 0}tgx-x=0, \: \lim_{x\to 0}x-sinx=0$ nên áp dụng định lý L'hôpital

$\lim_{x\to 0}\frac{tgx-x}{x-sinx}=\lim_{x\to 0}\frac{\frac{1}{cos^2x}-1}{1-cosx}=\lim_{x\to 0}\frac{1+cosx}{cos^2x}=2$

 

   2.2. Công thức Taylor và Maclaurin

Nếu hàm $y=f(x)$ khả vi $n+1$ lần thứ lân cận của điểm $x_0$ thì trong lân cận của $x_0$, ta có công thức:

$f(x)=f(x_0)+\frac{f'(x_0)}{1!}(x-x_0)+\frac{f''(x_0)}{2!}(x-x_0)^2+...+\frac{f^{(n)}(x_0)}{n!}(x-x_0)^n+R_n(x)$

gọi là công thức Taylor cấp $n$

Với $R_n(x)=\frac{f^{(n+1)}(c\: )}{(n+1)!}(x-x_0)^{(n+1)}$ là số hạng dư dạng Lagrange

 

Đặc biệt $x_0=0$ thì $f(x)=f(0)+\frac{f'(x_0)}{1!}x+\frac{f''(x_0)}{2!}x^2+...+\frac{f^{(n)}(x_0)}{n!}x-^n+R_n(x)$

gọi là công thức Maclaurin cấp $n$

 

 $*)$ Các khai triển thông dụng theo công thức Maclaurin:

 

$a)\: e^x=1+\frac{x}{1!}+\frac{x^2}{2!}+...+\frac{x^n}{n!}+\frac{e^{\theta x}\: x^{n+1}}{(n+1)!}$

 

$b)\: sinx=x-\frac{x^3}{3!}+\frac{x^5}{5!}+...+(-1)^{m-1}\frac{x^{2m-1}}{(2m-1)!}+...+sin(\frac{n\pi}{2})\: \frac{x^n}{n!}+sin[\theta x+(n+1)\frac{\pi}{2}]\:\frac{x^{n+1}}{(n+1)!}$

 

$c)\: cosx=1-\frac{x^2}{2!}+\frac{x^4}{4!}+...+(-1)^{m}\frac{x^{2m}}{(2m)!}+...+cos(\frac{n\pi}{2})\: \frac{x^n}{n!}+cos[\theta x+(n+1)\frac{\pi}{2}]\:\frac{x^{n+1}}{(n+1)!}$

 

$d)\: (1+x)^\alpha =1+\alpha x+\frac{\alpha (\alpha -1)}{2!}x^2+...+\frac{\alpha (\alpha -1)(\alpha -2)...(\alpha -n+1)}{n!}x^n+\frac{\alpha (\alpha -1)(\alpha -2)...(\alpha -n)}{(n+1)!}(1+\theta x)^{\alpha -n-1}x^{n+1},\: \: \alpha \: \epsilon \: \mathbb{R},\: 0<\theta<1.$

 

$e)\: ln(1+x)=x-\frac{x^2}{2}+...+(-1)^{n-1}\frac{x^n}{n}+\theta x^n$

 

$g)\: arctgx=\sum_{k=0}^{n}(-1)^{k}\frac{x^{2k+1}}{(2k+1)}+\theta (x^{2k+2})$

 

$h)\: arcsinx=x+\sum_{k=1}^{n}\frac{(2k-1)!!}{2^k\: k!\: (2k+1)}x^{2k+1}+\theta (x^{2k+2})$  




#447690 ĐẠO HÀM- VI PHÂN- TÍCH PHÂN- ỨNG DỤNG

Đã gửi bởi Mrnhan on 04-09-2013 - 12:38 trong Giải tích Toán học

                                                            CHƯƠNG II: THÍCH PHÂN

I. Tích phân bất định

1. khái niệm

1.1. Nguyên hàm

$F(x)$ gọi là một nguyên hàm của $f()x$ trong miền nếu $F'(x)=f(x)$ hay $dF(x)=f(x)\: \forall x\: \epsilon \: X$

1.2. Tích phân bất định của $f(x)$ trong $X$

$$\int f(x)dx=F(x)+C$$

Với $F(x)$ là một nguyên hàm của $f(x)$ và $C=const$

1.3. Tính chất

$a)\: \int [f(x)\pm g(x)]dx=\int f(x)dx \pm\int g(x)dx$

$b)\: \int Cf(x)dx=C\int f(x)dx$

$c)\: \int f(u)du=F(u)+C, \:\: u=u(x)$

VD: Tính tích phân

$\int \frac{1}{cos^6x}dx=tgx+\frac{2tg^3x}{3}+\frac{tg^5}{5}+C$

2. tích phân các hàm hữu tỉ

2.1 phương pháp chung

Hàm hữu tỉ có dạng: $f(x)=\frac{P_n(x)}{Q_m(x)}$

Với $P_n(x), \: Q_m(x)$ là các đa thức bậc $n, \:m$ của $x$

$n<m$: $f(x)$ là hàm hữu tỉ thật sự

$n\geq m$: $f(x)$ là hàm hữu tỉ không thực sự

Nếu $n<m$, $Q_m(x)=A(x-a)^\alpha...(x^2+px+q)^\beta,$ $(p^2-4q<0)$ thì:

$f(x)=\frac{A_1}{(x-a)^\alpha}+\frac{A_2}{(x-a)^{\alpha-1}}+...+\frac{A_\alpha}{x-a}+...+\frac{B_1x+C_1}{(x2px+q)^\beta}+\frac{B_2x+C_2}{(x^2+px+q)^{\beta-1}}+...+\frac{B_\beta x+C_\beta}{x^2+px+q}$

Qui đồng mãu số ở 2 vế rồi bỏ mẫu số, ta được hai đa thức dồng nhất nhau, cho bằng nhau các hệ số cùng lũy thừ của $x$ ở 2 vế, ta được 1 hệ phương trình để xác định:

$\left\{\begin{matrix}A_i, i=1, 2,..., \alpha\\B_i,\: C_i, \: i=1, 2, ..., \beta\end{matrix}\right.$

Khi đó $\int f(x)dx$ là tổng các tích phân các phân thức đơn gián ở vế phải.

2.2. Phương pháp Ostrogradski

Nếu $f(x)=\frac{P(x)}{Q(x)}$ và nếu $Q(x)$ có nghiêm bội: 

thì $I=\int f(x)dx=\int \frac{P(x)}{Q(x)}dx= \frac{X(x)}{Q_1(x)}dx+\int \frac{Y(x)}{Q_2(x)}dx$

Trong đó $Q_1(x)$ là ước số chung lớn nhất của $Q(x)$ và $Q'(x)$, còn $Q_2(x)=Q(x):Q_1(x)$.

$X(x), Y(x)$ là các đa thức hệ số chưa xác định, bậc của chúng kém bậc của $Q_1(x), \: Q_2(x)$ tương ứng một đơn vị. Các hệ số $X(x).\: Y(x)$ được xác định bằng các đạo hàm 2 vế, ta sẽ được 2 đa thức đồng nhất nhau, cho bằng nhau các hệ số cùng lũy thừa của $x$ ở 2 vế, ta sẽ được một hệ phương trình để xác định cá hệ số đó.

VD: Tính tích phân:

$$I=\int \frac{1}{(1+x^4)^2}$$

Giải:

$Q_1(x)=Q_2(x)=1+x^4$

Do đó: 

$I=\int \frac{1}{(1+x^4)^2}dx=\frac{Ax^3+Bx^2+Cx+D}{1+x^4}+\int\frac{Ex^3+Fx^2+Gx+H}{1+x^4}$

Đạo hàm 2 vế theo x và đồng nhất các hệ số của cùng lũy thừa ở 2 vế, ta có:

$1=(3Ax^2+2Bx+C)(1+x^4)-4x^3(Ax^3+Bx^2+Cx+D)+(1+x^4)(Ex^3+Fx^2+Gx+H)\to \left\{\begin{matrix} A=B=D=E=F=G=0\\C=\frac{1}{4},\: H=\frac{3}{4}\end{matrix}\right.$

$\to I=\frac{x}{4(1+x^4)}+\int \frac{3x}{4(1+x^4)}dx=\frac{x}{4(1+x^4)}+\frac{3}{16\sqrt{2}}ln(\frac{x^2+x\sqrt2+1}{x^2-x\sqrt2+1})+\frac{3}{8\sqrt2}arctg(\frac{x\sqrt2}{1-x^2})+C$

 

II. TÍCH PHÂN XÁC ĐỊNH

1. Khái niệm cơ bản

1.1. Định nghĩa

Tích phân xác định của hàm $f(x)$ xác định và bị chặn trên đoạn $[a;b]$:

$\int _{a}^{b} f(x)dx=\lim_{max \Delta x_i \to 0}\sum_{i=1}^{n}f(\zeta _i)\Delta x_i=\lim_{max \Delta x_i \to 0}I_n$

Với một cách chia bất kỳ đoạn $[a;b]$

$a=x_1<x_2<x_3<...<x_i<x_{i+1}<...<x_{n+1}=b \:\:$ 

$\Delta x_i=x_{i+1}-x_1, \:\:\zeta _i\:\epsilon \:[x_i;x_{i+1}], \:\:(i=\bar{1,n})$

$I_n$ gọi là tổng tích phân thứ $n$ của $f(x)$ trên $[a;b]$.

$f(x)$ có tích phân trên $[a;b]$ gọi là khả tích trên đó

1.2. Điều kiện khả tích (Riemann)

Đièu kiện cần và đủ để hàm bị chặn $f(x)$ khả tích trên đoạn $[a;b]$ là: $\lim_{max \Delta x_i \to 0}(S-s)=0$

Trong đó: $S=\sum_{i=1}^{n}M_i\Delta x_i, \:\:s=\sum_{i=1}^{n}m_i\Delta x_i$

$M_i=\sup_{[x_i;x_{i+1}]} \:f(x), \: m=\inf_{[x-i;x_{i+1}]}\:f(x), \: \:i=\bar{1,n}$

$S(s)$ gọi là các tổng Darboux trên (dưới ) của $f(x)$ ứng với cách chia nào đó của $[a;b]$, gọi chung là tổng Darboux.

Từ điều kiện khả tích, suy ra:

a) Mọi hàm $f(x)$ liên tục trên $[a;b]$ đều khả tích trên đoạn đó.

b) Mọi hàm $f(x)$ bị chặn và có một số hữu hạn điểm gián đoạn trên $[a;b]$ đều khả tích trên đoạn đó.

c) Mọi hàm $f(x)$ đơn điệu và bị chặn tên $[a;b]$ đều khả tích trên đoạn đó.

1.3. Ý nghĩ hình học và cơ học

$I=\int_{a}^{b}f(x)dx$, với $f(x)\geq 0$ trên đoạn $[a;b]$ là diện tích hình thang cong giới hạn bởi các đường thẳng $x=a,\:x=b,\:y=0$ và $y=f(x)$

2.png

$I=\int_{a}^{b}f(x)dx$, với $f(x)>0$ trên đoạn $[a;b]$ là:

- Khối lượng của đoạn $[a;b]$ với mật độ khối lượng (dài )$f(x)$

- Công của lực có đọ lớn $f(x)>0$ tác dụng vào một vật chuyển động thẳng từ $x=a\to x=b$.

1.4. Tính chất

$a)\: \int_{a}^{b} [f(x)\pm g(x)]dx=\int_{a}^{b} f(x)dx \pm\int_{a}^{b} g(x)dx$

$b)\: \int_{a}^{b} Cf(x)dx=C\int_{a}^{b} f(x)dx$

$c)\: \int_{a}^{b} f(u)du=F(u)+C, \:\: u=u(x)$

2. Đạo hàm theo cận trên- Công thức Newton-Leibniz- Phương pháp tính cơ bản

2.1 Đạo hàm theo cận trên

- Nếu $F(x)$ khả tích trên $[a;b]$ thì $I(x)=\int_{a}^{x}f(t)dt$ là một hàm liên tục trên $[a;b]$

- Nếu thêm điều kiện $f(t)$ liên tục tại $t=x\:\epsilon\:[a;b]$ thì $I'(x)=f(t)$

- Mọi hàm liên tục trên $[a;b]$ đều có nghuyên hàm trên đoạn đó.

2.2. Công thức Newton-Leibniz

Nếu $f(x)$ liên tục trên $[a;b]$ thì $\int_{a}^{b}f(x)dx=F(x)|_{a}^{b}=F(b)-F(a)$

Với $F(x)$ là một nguyên hàm của $f(x)$ trên $[a;b].$

2.3. Phương pháp tính tích phân.

- Phương pháp từng phần:

Nếu các hàm $u(x)\: v(x)$ khả vi liên tục trên đoạn $[a;b]$ thì:

$\int_{a}^{b}udv=(uv)|_{a}^{b}-\int_{a}^{b}vdu$

- Phương pháp đỏi biến: 

Để tính $I=\int_{a}^{b}f(x)dx$, $f(x)$ liên tục trên $[a;b]$, đặt $x=x(t),\: \alpha\leq t\leq\beta$, thỏa mãn:

$x(t)$ và $x'(t)$ liên tục trong $[\alpha; \beta]$, $\alpha\leq t\leq\beta\to a\leq x\leq b$.

$x(\alpha)=a;\: x(\beta)=b$ thì $\int_{a}^{b}f(x)dx=\int_{\alpha}^{\beta}f(x(t))x'(t)dt$

3. ỨNG DỤNG CỦA TÍCH PHÂN

3.1. Tính diện tích mặt phẳng

Diện tích $S$ của hình thang cong giới hạn bởi các đường lên tục $y=f(x), \: x=a, \: x=b$ và trục $Ox$

$S=\int_{a}^{b}|f(x)|dx$

tt xoay.png

Nếu hình giới hạn bởi $a\leq x\leq b,\: y_1\leq y\leq y_2$ thì:

$S=\int_{a}^{b}(y_2-y_1)dx$

Nếu đường thẳng $y=f(x)$ cho dưới dang tham số $x=x(t), \: y=y(t), \: \alpha \leq t \leq \beta$ ứng với $a\leq x\leq b$ thì:

$S=\int_{a}^{b}|y(t)x'(t)|dt$

Nếu đường thẳng  $x=x(t),\: y=y(t), 0\leq t\leq T$ là đường kín liên tục, chạy ngược kim đồng hồ và dưới hạn diện tích $S$ và phía trái thì:

$S=-\int_{0}^{T}y(t)x'(t)dt=\int_{0}^{T}x(t)y'(t)dt\to S=\frac{1}{2 }[\int_{0}^{T}x(t)y'(t)-\int_{0}^{T}y(t)x'(t)]dt$

Trong tọa độ độc cực, diện tích $S$ của hình giới hạn bởi các tia: $\varphi =\alpha,\: \varphi =\beta$ và đường thẳng $r=r(\varphi )$ là:

$S=\frac{1}{2}\int_{\alpha}^{\beta}r^2(\varphi )d\varphi$

3.2. Tính độ dài đường cong

Độ dài $s$ của cung đường cong $\widehat{AB}:\: y=y(x),\: y'(x)$ liên tục, $a\leq x\leq b:$

$s=\int_{a}^{b}\sqrt{1+y'^2_x}dx$

Nếu  $\widehat{AB}$ có phương trình tham số $x=x(t), \: y=y(t), \: \alpha \leq t \leq \beta$ ứng với $a\leq x\leq b$ thì:

$s=\int_{\alpha }^{\beta }\sqrt{x'^2_t+y'^2_t}\: dt$

Trong tọa độ cực dài của $AB$: $r=r(\varphi )$$\alpha \leq \varphi \leq \beta:$

$s=\int_{\alpha }^{\beta }\sqrt{r^2+r'^2_\varphi}\: d\varphi$

3.3. Tính thể tích

3.3.1. Thể tích các khối khi quay quanh $Ox$

Thể tích $V_x$ của khối tròn xoay sinh bởi diện tích $S$ giới hạn $y=f(x)$, trục $Ox$, 2 đường $y==f(x),\: y=g(x)$ các cận $x=a,\: x=b$ khi quay quanh trục $Ox$ là dấu hiệu của thể tích bên ngoài trừ đi thể tích phần lõi ở bên trong. Giả sử $)<g(x)<f(x)$ trong $[a;b]$ thì ta có:

$V_x=\pi\int_{a}^{b}[f^2(x)-g^2(x)]dx$

Trong trường hợp $f(x)$ và $g(x)$ tự cắt nhau vf tạo thành hình khép kín, ta phải tìm giao điểm để biết cận lấy tích phân, rồi lấy thể tích khối bên ngoài trừ đi thể tích khối lõi bên trong

Thể tích $V_x$ của khối tròn xoay sinh bởi diện tích $S$ do q đường cong bậc 2 tạo ra. Ta tách đường cong thành 2 hàm theo đôi số $x$, khi đó ta coi $S$ giới hạn bởi:

$\left\{\begin{matrix}(C_1)y=f_1(x)\\(C_2)y=f_2(x)\end{matrix}\right.$

Nên ta có $V_x=\pi\int_{a}^{b}|f^2_1(x)-f^2_2(x)|dx$

tt xoay 3.png

3.3.2. Thể tích các khối khi quay quanh $Oy$

Thể tích $V_y$ sinh ra do $S$ giới hạn bởi $y=f(x),\: Oy,\: y=m,\: y=n$ khi quay quanh $Oy$

$V_y=\pi \int_{m}^{n}[f^{-1}(x)]^2dy$

Thể tích $V_y$ sinh ra do $S$ giới hạn bởi $y=f(x),\: y=g(x), y=m,\:y=n$ khi quay quanh $Oy$ là $V_y=\int_{m}^{n}[(f^{-1}(x))^2-(g^{-1}(x))^2]dy$

Thể tích $V_y$ sinh ra do $S$ giới hạn bởi đường cong bậc 2: $f(x,y)=0$. Tách đường cong thành 2 đường

$\left\{\begin{matrix}(C_1)y=f_1(x)\\(C_2)y=f_2(x)\end{matrix}\right.$

Thể tích cần tìm khi đó $V_y=\pi \int_{m}^{n}|[f_1^{-1}(x)]^2-[f_2^{-1}(x)]^2|dy$

3.png

4. TÍCH PHÂN MỞ RỘNG

A. Tích phân suy rộng có cận vô cực

Nếu hàm số $y=f(x)$ xác định trên $[a;b]$ vô hạn và khả tích trên mỗi đoạn hữu hạn $-\infty<a\leq x\leq b<+\infty$

1.1. $\int_{-\infty}^{b}f(x)dx=\lim_{a\to -\infty}\int_{a}^{b}f(x)dx$

 

1.2. $\int_{a}^{+\infty}f(x)dx=\lim_{b\to +\infty}\int_{a}^{b}f(x)dx$

 

1.3. Tổng quát: $\int_{-\infty}^{+\infty}f(x)dx=\lim_{a\to -\infty, b\to +\infty}\int_{a}^{b}f(x)dx$

 

Nếu giới hạn $\lim_{a\to -\infty, b\to +\infty}\int_{a}^{b}f(x)dx$ là hữu hạn thì tích phân suy rộng $\int_{-\infty}^{+\infty}f(x)dx$ là hội tụ (integral is convergent ).

 

Ngược lại, nếu giới hạn $\lim_{a\to -\infty, b\to +\infty}\int_{a}^{b}f(x)dx$ là vô cùng hoặc không tồn tại thì tích phân suy rộng $\int_{-\infty}^{+\infty}f(x)dx$ là phân kỳ (integral is divergent ).

2. Ứng dụng

 

2.1. Chứng minh: $I=\int_{0}^{+\infty}cosxdx$ phân kỳ

 

2.2. Chứng minh: $I=\int_{1}^{+\infty}\frac{1}{x^2}dx$ hội tụ

B. Tích phân suy rộng của hàm không bị chặn

Nếu hàm số $y=f(x)$ xác định và khả tích mỗi đoạn trên $[a+\varepsilon ;b],\forall \varepsilon >0, \lim_{x\to a^{+}}f(x)=\infty$

 

$\int_{a}^{b}f(x)dx=\lim_{\varepsilon\to 0^+}\int_{a+\varepsilon}^{b}f(x)dx$

 

Nếu hàm số $y=f(x)$ xác định và khả tích mỗi đoạn trên $[a ;b-\varepsilon],\forall \varepsilon >0, \lim_{x\to b^{-}}f(x)=\infty$

 

$\int_{a}^{b}f(x)dx=\lim_{\varepsilon\to 0^-}\int_{a}^{b-\varepsilon}f(x)dx$

 

Nếu hàm số $y=f(x)$ có điểm gián đoạn tại $c$ trên đoạn $[a ;b],\:$$ \lim_{x\to c}f(x)=\infty$ và liên tục khi $a\leq x<c, c<x\leq b$
 
$\int_{a}^{b}f(x)dx=\lim_{\varepsilon _1\to o^-}\int_{a}^{c-\varepsilon _1}f(x)dx+\lim_{\varepsilon _2\to o^+}\int_{c+\varepsilon _2}^{b}f(x)dx$

 




#445991 $\int_{0}^{1}\sqrt{x^2+1}dx$

Đã gửi bởi Mrnhan on 28-08-2013 - 20:51 trong Tích phân - Nguyên hàm

Tính: $\int_{0}^{1}\sqrt{x^2+1}dx$

 

MOD : Chú ý Latex

CK: $x=tant\to dx=\frac{dt}{cos^2t}$

$I=\int \frac{dt}{cos^3t}=\frac{d(sint)}{(1-sin^2t)^2}....$




#432290 Phương trình, Bất phương trình, Hệ phương trình

Đã gửi bởi Mrnhan on 02-07-2013 - 14:06 trong Ôn thi Đại học



Bài 14
$\left\{\begin{matrix} x^4+x^3y+x^2x^2+xy^3+y^4=5 & \\ 2\sqrt{1-x^2}+\sqrt{2x^2-y^2}=2 & \end{matrix}\right.$


 

 

Giai:

Ap dung BĐT Bunhia: $4=(\sqrt{2}\sqrt{2-2x^2}+\sqrt{2x^2-y^2})^2\leq 3(2-y^2)\to y<1$

Tu PT $\to x<1$

$\to x^4+x^3y+x^2x^2+xy^3+y^4<5\to$ PTVN
 




#432288 Phương trình, Bất phương trình, Hệ phương trình

Đã gửi bởi Mrnhan on 02-07-2013 - 13:59 trong Ôn thi Đại học

còn bài này thì ai có cách làm khác cách của nthoangcute ko? Chỉ cho mình với.

 

Bài 13

$\left\{\begin{matrix} 2x^3-xy^2=1 & \\ 2x^2-3xy+2y^2=x-y & \end{matrix}\right.$

(Đề thi thử của trung tâm luyện thi Tô hoàng)

Giải:

$x^2-(3y+1)x+2y^2+y=0$$x^2-(3y+1)x+2y^2+y=0$

Xét $\Delta =(3y+1)^2-4(2y^2+y)=(y+1)^2$

Thay vào giai binh thuong...!!




#432306 Phương trình, Bất phương trình, Hệ phương trình

Đã gửi bởi Mrnhan on 02-07-2013 - 15:06 trong Ôn thi Đại học


Bài 17
$\left\{\begin{matrix} xy+\sqrt{2(x^4+y^4)}=1 & \\ x^{2009}y^{2013}+x^{2013}y^{2009}=\frac{2}{3^{2011}} & \end{matrix}\right.$


 

PT1: $xy+\sqrt{2(x^4+y^4)}=1\Leftrightarrow x^4+y^4=\frac{(1-xy)^2}{2}$ và $xy\leq \frac{1}{3}$

PT2: $\frac{2}{3^{2011}}=(xy)^{2009}(x^4+y^4)=(xy)^{2009}.\frac{(1-xy)^2}{2}=2(xy)^{2008}(xy)\frac{1-xy}{2}.\frac{1-xy}{2}\leq 2.(xy)^{2008}.(\frac{xy+1-xy}{3})^3=\frac{2(xy)^{2008}}{3^3}\leq \frac{2}{3^{2011}}$

Vay $x=y=\pm \frac{1}{\sqrt{3}}$




#459708 Tìm hạng của ma trận phụ thuộc tham số

Đã gửi bởi Mrnhan on 24-10-2013 - 19:32 trong Đại số tuyến tính, Hình học giải tích



Biểu thức cuối cùng: $\begin{vmatrix} 1 &3 &-2 &k \\ 0 &-4 & k+2 &3-k \\ 0 &0 &-2k-9 &4k-9 \end{vmatrix}$

Ở đây nếu chọn: $D_{12}^{12}=\begin{vmatrix} 1 &3 \\ 0 &-4 \end{vmatrix}=-4\neq 0$$D_{12}^{12}=\begin{vmatrix} 1 &3 \\ 0 &-4 \end{vmatrix}=-4\neq 0$

Tính tiếp định thức cấp 3 bao quanh nó:

$D_{123}^{123}=\begin{vmatrix} 1 &3 &-2 \\ 0 &-4 &k+2 \\ 0 & 0 &-2k-9 \end{vmatrix}=4(2k+9)$

Đến đây chỉ có 2 trường hợp: $k\neq -\frac{9}{2}$ hoặc $k=-\frac{9}{2}$

Nếu tính định thức bao quanh khác thì lại ra một giá trị k khác, lúc đó thì biện luận kiểu gì ạ?

Dạ, em ghi thế thôi, chứ em có biết chi mu?

 

Mà chị nên đưa bài ra ngoài hỏi!

 



$A=\begin{pmatrix} 1 & 3& -2& k\\ 1 & -1& k& 3\\ -2 & 2& -1& -3\end{pmatrix}$

 

$\to\begin{pmatrix} 1 & 3& -2& k\\ 0 & -4& k+2& 3-k\\ 0 & 8& -5& 2k-3\end{pmatrix}$

 

$\to \begin{pmatrix} 1 & 3& -2& k\\ 0 & -4& k+2& 3-k\\ 0 & 0& -2k-9& 4k-9\end{pmatrix}$

 

Từ đó biện luận và kết luận là $rank(A)=3$

 

P/s: @vo van duc: Anh có thể rời bài này ra ngoài!

 

Quên, em gõ nhầm mất!!

 

Em biện luận như sau:

 

$\begin{vmatrix} k+2& 3-k\\ -2k-9& 4k-9\end{vmatrix}=2k^2-4k+9>0$

 

$\begin{vmatrix} 3& -2& k\\ -4& k+2& 3-k\\ 0& -2k-9& 4k-9\end{vmatrix}=14k^2-8k+99>0$

 

Không cần vòng nữa!

 

Nên với mọi $k$ thì $rank(A)=3$